Does bounded version of Lagrange's 4-square theorem follow from this result?












0












$begingroup$


Let $S_4(n)$ be the number of ways of representing $n$ as a sum of 4 integer squares and suppose I can give you a proof of the result that $sum_{i=1}^nS_4(i) sim frac{pi^2}{2}n^2$.



Is this result enough to deduce that beyond some $N$, every integer can be represented as the sum of 4 squares (given that we are "weakly" suggesting that $sum_{i=1}^nS_4(i) >> n$)?










share|cite|improve this question











$endgroup$








  • 1




    $begingroup$
    It's not true that $S_4(n)simfrac{pi^2}{2}n^2$. What is true is that $sum_{k=1}^nS_4(k)simfrac{pi^2}{2}n^2$.
    $endgroup$
    – Lord Shark the Unknown
    Dec 15 '18 at 16:16










  • $begingroup$
    Ah yes, I meant that! Let me just update this.
    $endgroup$
    – Isky Mathews
    Dec 15 '18 at 17:12










  • $begingroup$
    @LordSharktheUnknown: Can you answer the question, though? I know from experience on this site that you're quite a capable individual...
    $endgroup$
    – Isky Mathews
    Dec 15 '18 at 17:13










  • $begingroup$
    You need more than that asymptotic.
    $endgroup$
    – Lord Shark the Unknown
    Dec 15 '18 at 19:13
















0












$begingroup$


Let $S_4(n)$ be the number of ways of representing $n$ as a sum of 4 integer squares and suppose I can give you a proof of the result that $sum_{i=1}^nS_4(i) sim frac{pi^2}{2}n^2$.



Is this result enough to deduce that beyond some $N$, every integer can be represented as the sum of 4 squares (given that we are "weakly" suggesting that $sum_{i=1}^nS_4(i) >> n$)?










share|cite|improve this question











$endgroup$








  • 1




    $begingroup$
    It's not true that $S_4(n)simfrac{pi^2}{2}n^2$. What is true is that $sum_{k=1}^nS_4(k)simfrac{pi^2}{2}n^2$.
    $endgroup$
    – Lord Shark the Unknown
    Dec 15 '18 at 16:16










  • $begingroup$
    Ah yes, I meant that! Let me just update this.
    $endgroup$
    – Isky Mathews
    Dec 15 '18 at 17:12










  • $begingroup$
    @LordSharktheUnknown: Can you answer the question, though? I know from experience on this site that you're quite a capable individual...
    $endgroup$
    – Isky Mathews
    Dec 15 '18 at 17:13










  • $begingroup$
    You need more than that asymptotic.
    $endgroup$
    – Lord Shark the Unknown
    Dec 15 '18 at 19:13














0












0








0





$begingroup$


Let $S_4(n)$ be the number of ways of representing $n$ as a sum of 4 integer squares and suppose I can give you a proof of the result that $sum_{i=1}^nS_4(i) sim frac{pi^2}{2}n^2$.



Is this result enough to deduce that beyond some $N$, every integer can be represented as the sum of 4 squares (given that we are "weakly" suggesting that $sum_{i=1}^nS_4(i) >> n$)?










share|cite|improve this question











$endgroup$




Let $S_4(n)$ be the number of ways of representing $n$ as a sum of 4 integer squares and suppose I can give you a proof of the result that $sum_{i=1}^nS_4(i) sim frac{pi^2}{2}n^2$.



Is this result enough to deduce that beyond some $N$, every integer can be represented as the sum of 4 squares (given that we are "weakly" suggesting that $sum_{i=1}^nS_4(i) >> n$)?







number-theory proof-verification asymptotics sums-of-squares






share|cite|improve this question















share|cite|improve this question













share|cite|improve this question




share|cite|improve this question








edited Dec 15 '18 at 21:39







Isky Mathews

















asked Dec 15 '18 at 16:13









Isky MathewsIsky Mathews

903314




903314








  • 1




    $begingroup$
    It's not true that $S_4(n)simfrac{pi^2}{2}n^2$. What is true is that $sum_{k=1}^nS_4(k)simfrac{pi^2}{2}n^2$.
    $endgroup$
    – Lord Shark the Unknown
    Dec 15 '18 at 16:16










  • $begingroup$
    Ah yes, I meant that! Let me just update this.
    $endgroup$
    – Isky Mathews
    Dec 15 '18 at 17:12










  • $begingroup$
    @LordSharktheUnknown: Can you answer the question, though? I know from experience on this site that you're quite a capable individual...
    $endgroup$
    – Isky Mathews
    Dec 15 '18 at 17:13










  • $begingroup$
    You need more than that asymptotic.
    $endgroup$
    – Lord Shark the Unknown
    Dec 15 '18 at 19:13














  • 1




    $begingroup$
    It's not true that $S_4(n)simfrac{pi^2}{2}n^2$. What is true is that $sum_{k=1}^nS_4(k)simfrac{pi^2}{2}n^2$.
    $endgroup$
    – Lord Shark the Unknown
    Dec 15 '18 at 16:16










  • $begingroup$
    Ah yes, I meant that! Let me just update this.
    $endgroup$
    – Isky Mathews
    Dec 15 '18 at 17:12










  • $begingroup$
    @LordSharktheUnknown: Can you answer the question, though? I know from experience on this site that you're quite a capable individual...
    $endgroup$
    – Isky Mathews
    Dec 15 '18 at 17:13










  • $begingroup$
    You need more than that asymptotic.
    $endgroup$
    – Lord Shark the Unknown
    Dec 15 '18 at 19:13








1




1




$begingroup$
It's not true that $S_4(n)simfrac{pi^2}{2}n^2$. What is true is that $sum_{k=1}^nS_4(k)simfrac{pi^2}{2}n^2$.
$endgroup$
– Lord Shark the Unknown
Dec 15 '18 at 16:16




$begingroup$
It's not true that $S_4(n)simfrac{pi^2}{2}n^2$. What is true is that $sum_{k=1}^nS_4(k)simfrac{pi^2}{2}n^2$.
$endgroup$
– Lord Shark the Unknown
Dec 15 '18 at 16:16












$begingroup$
Ah yes, I meant that! Let me just update this.
$endgroup$
– Isky Mathews
Dec 15 '18 at 17:12




$begingroup$
Ah yes, I meant that! Let me just update this.
$endgroup$
– Isky Mathews
Dec 15 '18 at 17:12












$begingroup$
@LordSharktheUnknown: Can you answer the question, though? I know from experience on this site that you're quite a capable individual...
$endgroup$
– Isky Mathews
Dec 15 '18 at 17:13




$begingroup$
@LordSharktheUnknown: Can you answer the question, though? I know from experience on this site that you're quite a capable individual...
$endgroup$
– Isky Mathews
Dec 15 '18 at 17:13












$begingroup$
You need more than that asymptotic.
$endgroup$
– Lord Shark the Unknown
Dec 15 '18 at 19:13




$begingroup$
You need more than that asymptotic.
$endgroup$
– Lord Shark the Unknown
Dec 15 '18 at 19:13










0






active

oldest

votes











Your Answer





StackExchange.ifUsing("editor", function () {
return StackExchange.using("mathjaxEditing", function () {
StackExchange.MarkdownEditor.creationCallbacks.add(function (editor, postfix) {
StackExchange.mathjaxEditing.prepareWmdForMathJax(editor, postfix, [["$", "$"], ["\\(","\\)"]]);
});
});
}, "mathjax-editing");

StackExchange.ready(function() {
var channelOptions = {
tags: "".split(" "),
id: "69"
};
initTagRenderer("".split(" "), "".split(" "), channelOptions);

StackExchange.using("externalEditor", function() {
// Have to fire editor after snippets, if snippets enabled
if (StackExchange.settings.snippets.snippetsEnabled) {
StackExchange.using("snippets", function() {
createEditor();
});
}
else {
createEditor();
}
});

function createEditor() {
StackExchange.prepareEditor({
heartbeatType: 'answer',
autoActivateHeartbeat: false,
convertImagesToLinks: true,
noModals: true,
showLowRepImageUploadWarning: true,
reputationToPostImages: 10,
bindNavPrevention: true,
postfix: "",
imageUploader: {
brandingHtml: "Powered by u003ca class="icon-imgur-white" href="https://imgur.com/"u003eu003c/au003e",
contentPolicyHtml: "User contributions licensed under u003ca href="https://creativecommons.org/licenses/by-sa/3.0/"u003ecc by-sa 3.0 with attribution requiredu003c/au003e u003ca href="https://stackoverflow.com/legal/content-policy"u003e(content policy)u003c/au003e",
allowUrls: true
},
noCode: true, onDemand: true,
discardSelector: ".discard-answer"
,immediatelyShowMarkdownHelp:true
});


}
});














draft saved

draft discarded


















StackExchange.ready(
function () {
StackExchange.openid.initPostLogin('.new-post-login', 'https%3a%2f%2fmath.stackexchange.com%2fquestions%2f3041664%2fdoes-bounded-version-of-lagranges-4-square-theorem-follow-from-this-result%23new-answer', 'question_page');
}
);

Post as a guest















Required, but never shown

























0






active

oldest

votes








0






active

oldest

votes









active

oldest

votes






active

oldest

votes
















draft saved

draft discarded




















































Thanks for contributing an answer to Mathematics Stack Exchange!


  • Please be sure to answer the question. Provide details and share your research!

But avoid



  • Asking for help, clarification, or responding to other answers.

  • Making statements based on opinion; back them up with references or personal experience.


Use MathJax to format equations. MathJax reference.


To learn more, see our tips on writing great answers.




draft saved


draft discarded














StackExchange.ready(
function () {
StackExchange.openid.initPostLogin('.new-post-login', 'https%3a%2f%2fmath.stackexchange.com%2fquestions%2f3041664%2fdoes-bounded-version-of-lagranges-4-square-theorem-follow-from-this-result%23new-answer', 'question_page');
}
);

Post as a guest















Required, but never shown





















































Required, but never shown














Required, but never shown












Required, but never shown







Required, but never shown

































Required, but never shown














Required, but never shown












Required, but never shown







Required, but never shown







Popular posts from this blog

Plaza Victoria

In PowerPoint, is there a keyboard shortcut for bulleted / numbered list?

How to put 3 figures in Latex with 2 figures side by side and 1 below these side by side images but in...